LSAT and Law School Admissions Forum

Get expert LSAT preparation and law school admissions advice from PowerScore Test Preparation.

 Administrator
PowerScore Staff
  • PowerScore Staff
  • Posts: 8916
  • Joined: Feb 02, 2011
|
#36480
Complete Question Explanation

Strengthen—#%. The correct answer choice is (D)

As is often the case with Numbers and Percentages questions, you should beware of situations in
which limited information is provided, and the author jumps to a few conclusions. In this example,
looking at address changes that have been filed at the post office over the last decade, twice as many
households have reported leaving the city as entering. The stimulus’ author concludes that next year’s
census will reflect a population decline over the past ten years, since the last census was taken.
  • Premise: Twice as many households are leaving the city as are entering (according to
    change of address forms filed at the post office over the past ten years)

    Conclusion: The next census taken will surely reflect a population decline having taken
    place over the last ten years.
The author’s argument is not without a few holes—for example, we can’t be sure how many people
were in each reporting household. The stimulus also fails to provide what portion of movers filed
with the post office, so there is no way to gauge the reliability of the data used as the basis of the
author’s conclusion, or how many births and deaths have occurred over the last ten years.

Since the stimulus is followed by a Strengthen question, the correct answer choice will provide
evidence that makes it more likely that the city’s population has indeed declined over the last decade.
With all else equal, any information that would tend to yield the predicted decrease in population
will strengthen the author’s argument.

Answer choice (A): The term “many” is quite vague, and this choice neither suggests a net gain nor
a net loss in the city’s population. This choice should be ruled out, because it neither strengthens nor
weakens the author’s claim that the population will be shown to have decreased over the past decade.

Answer choice (B): No information is provided regarding the census before the last one, so there
is no way to know whether the last census showed an increase in population, a decrease, or neither.
The trend discussed in this choice would be of questionable relevance even if we did have such
information, but since that data is not provided, this choice does not address the facts provided in
the stimulus and can’t be the right answer.

Answer choice (C): First of all, the word “many” is not very helpful—is that three people? One
hundred? Second of all, the fact that many people entered the city “under the radar,” without filing
any paperwork with the post office, is not relevant, because the author’s claims concern the findings
of the census.

Answer choice (D): This is the correct answer choice, because it provides evidence that points
to a population decline: If most of the households moving out of town were families, this would
increase the flow out of town, driving down the population. And, if most of the households entering
or remaining in the city are older individuals, this strengthens the case for population decline as well.

Answer choice (E): “Many” is such a vague term that it provides almost no information about the
number of young adults leaving the city. The fact that most who stayed had long standing jobs is also
not very helpful—it doesn’t really matter why they stayed—just that they stayed.
 preppingaway
  • Posts: 1
  • Joined: Sep 23, 2015
|
#19937
I don't understand why (c) is incorrect. It seems to me like fewer reports of incoming people gives credence to the conclusion that that the census data shows a decrease in population.

Is the issue that we don't know if the peoples' tendency to report has actually decreased as opposed to always being low?
 Steve Stein
PowerScore Staff
  • PowerScore Staff
  • Posts: 1153
  • Joined: Apr 11, 2011
|
#19939
Hi Prepping Away!

That's a great question. In that one, looking at address changes that have been filed at the post office over the last decade, twice as many households have reported leaving the city as entering. The author concludes that next year’s census will reflect a ten-year population decline (since the last census was taken).
  • Premise: Twice as many households are leaving the city as are entering (according to change of address forms filed at the post office over the past ten years).
  • Conclusion: The next census taken will surely reflect a population decline having taken place over the last ten years.
The author’s argument is somewhat problematic—for example, we can’t be sure how many people were in each reporting household. The stimulus also fails to provide what portion of movers filed with the post office, so there is no way to determine the reliability of the data used as the basis of the author’s conclusion, or how many births and deaths have occurred over the last ten years.

Since the stimulus is followed by a Strengthen question, the correct answer choice will provide evidence that makes it more likely that the city’s population has indeed declined over the last decade (with all else equal, any information that would tend to yield the predicted decrease in population will strengthen the author’s argument).

Many students found answer choice (C) appealing, but that choice has a couple of issues. First of all, the word “many” is not very helpful—that could be three people, or 1000. Beyond that, the fact that many people entered the city “under the radar,” without filing any paperwork with the post office, is not relevant, because the author’s claims concern the findings of the census.

The right answer is answer choice (D), because it provides evidence that suggests a population decline: If most of the households moving out of town were families, this would increase the flow out of town, driving down the population. And, if most of the households entering or remaining in the city are older individuals, this would make a population decline more likely as well.

I hope that's helpful! Numbers and Percentage questions can be particularly challenging, so please let me know whether this is all clear—thanks!

~Steve

Get the most out of your LSAT Prep Plus subscription.

Analyze and track your performance with our Testing and Analytics Package.